Jump to content

limits.... gahh either I forgot or just no clue


caseclosed

Recommended Posts

heh, I just found out what I forgot, I forgot to balance out the e by adding natural log. so it would be e^L=e^(ln(e^x+x)*(1/x)) and then by using properties of log and change L to quotient to get (e^x+x)/x so that it is form infinity/infinity and now using L Hopital's rule to get (1+1)/1 so it is 2 but 2=L so the answer would be e^2. :) woot...

Link to comment
Share on other sites

  • 1 month later...

Let consider:

Lim x -> 0 {f(x)}^g(x), where as x -> 0 ,f(x) -> 1,and g(x) ->infinity

lim x->0 [f(x)]^g(x)=lim x->0[1+f(x)-1]^{{g(x)(f(x)-1)}/(f(x)-1)}

= e^lim x ->0{g(x)(f(x)-1)}

here ,we consider only the part; lim x->0 (e^x)^(1/x)

f(x)=e^x and g(x)=1/x so limit becomes :

e^[lim x->0 ((e^x-1)/x)=e^1=e :as (e^x-1)/x=1

Link to comment
Share on other sites

Create an account or sign in to comment

You need to be a member in order to leave a comment

Create an account

Sign up for a new account in our community. It's easy!

Register a new account

Sign in

Already have an account? Sign in here.

Sign In Now
×
×
  • Create New...

Important Information

We have placed cookies on your device to help make this website better. You can adjust your cookie settings, otherwise we'll assume you're okay to continue.